What is the volume of the shape below

What Is The Volume Of The Shape Below

Answers

Answer 1

The volume of the given figure is 1176 cubic cm.

The value of the figure is calculated by multiplying all three side areas. Volume is a measure of the amount of space that an object or a substance occupies. It is typically expressed in cubic units such as cubic meters (m³), cubic centimetres (cm³), or cubic feet (ft³).

The volume of the figure is calculated as,

Volume = 2 ( 15 x 6 + 20 x 6 + 20 x 18 )

Volume = 1176 Cubic cm

Hence, the volume of the figure will be equal to 1176 Cubic cm.

To know more about volume follow

https://brainly.com/question/29568631

#SPJ1


Related Questions

A virus takes 5 days to grow from 180 to 230. How many days will it take to grow from 180 to 260? Round to the nearest whole number.

Answers

It will take approximately 8 days for the virus to grow from 180 to 260.

We can set up a proportion to solve this problem. Let "x" represent the number of days it will take for the virus to grow from 180 to 260.

The proportion can be set up as follows;

(Change in value) / (Time taken) = (Change in value) / (Time taken)

Using the given information, we have;

(260 - 180) / x = (230 - 180) / 5

Simplifying the fractions on both sides of the equation, we get:

80 / x = 50 / 5

Cross-multiplying, we have;

80 × 5 = 50 × x

400 = 50x

Dividing both sides of the equation by 50, we get;

x = 400 / 50

x = 8

Therefore, it will take 8 days.

To know more about virus here

https://brainly.com/question/28067273

#SPJ1

Find the value of the following expression and round to the nearest integer:

Answers

The value of the Expression is 610, 919.

We have,

Expression : [tex]\sum_{n=0}^{61[/tex] 700 (1.07[tex])^{n+1[/tex]

The expression is a summation formula, representing the sum of a series of values.

Here r= 1.07 and n is number of terms.

So, [tex]\sum_{n=0}^{61[/tex] 700 (1.07[tex])^{n+1[/tex]

= 700 (1) +  [tex]\sum_{n=0}^{61[/tex] 700 (1.07[tex])^{n[/tex]

= 700 + 700 ([tex]r^{61[/tex] - 1)/ (r-1)

= 700 + 700 ([tex](1.07)^{61[/tex]-1)/ (0.07)

= 700+ 700 (871.7428)

= 610, 919

Learn more about Summation here:

https://brainly.com/question/29103782

#SPJ1

Please help asap!!!!! I'm confused

Answers

The area of the parallelogram is 8.5 square miles. This is found by multiplying the length of one of the parallel sides, 2 miles, by the height, which is given as 4 1/4 miles.

To find the area of a parallelogram, we can multiply the length of one of its parallel sides by the length of its perpendicular height. Therefore, to find the area of this parallelogram, we need to determine its height.

We are given that one of the parallel sides has a length of 4 1/4 mi and the other has a length of 2 mi. We are also given that the length of the perpendicular on one of the parallel sides is 4 1/4 mi, which means that this is the height of the parallelogram.

So, the area of the parallelogram is

Area = base x height

Area = 2 mi x 4 1/4 mi

Area = 8 1/2 mi²

Therefore, the area of the parallelogram is 8 1/2 square miles or 8.5 square miles.

To know more about Area of parallelogram:

https://brainly.com/question/19187448

#SPJ1

A group consists of seven Democrats and eight Republicans. Four people are selected to attend a conference.
a. In how many ways can four people be selected from this group of fifteen?
b. In how many ways can four Republicans be selected from the eight Republicans?
c. Find the probability that the selected group will consist of all Republicans.
a. The number of ways to select four people from the group of fifteen is
b. The number of ways to select four Republicans from the group of eight Republicans is
c. The probability is

Answers

There 1365 ways to choose four people from the group of fifteen.

b. There are 70 ways to choose  four Republicans from the group of eight Republicans.

C. The probability is about  0.0513, or  5.13%.

What is the probability  about?

a.  To know the ways that four people can be selected from this group of fifteen is by:

nCr = n! / (r! x (n-r)!),

Where:

n  = total number of items

r = is the number of items to be selected,

!  = the factorial of a number.

Putting in the values into the the formula:

15C4 = 15! / (4! x (15-4)!)

(15-4)! = 11!

15C4 = 1365

B.  Since:

n = 8

r = 4

Putting in the values into the the formula:

8C4 = 8! / (4!  x (8-4)!)

(8-4)! = 4!

8C4 = 70

c. The  Probability = Number of ways to choose four Republicans / Number of ways to choose four people

Hence  Probability = 70 / 1365

                             =  0.0513

Therefore, the probability that the selected group will consist of all Republicans is about  0.0513, or  5.13%.

Learn more about probability  from

https://brainly.com/question/24756209

#SPJ1

I need help as soon as possible please??

Answers

Answer:

4x² - 11xy - 3y²

Step-by-step explanation:

(4x + y)(x - 3y) =

Multiply each term of the first binomial by each term of the second binomial.

= 4x² - 12xy + xy - 3y²

Now combine like terms.

= 4x² - 11xy - 3y²

In parallelogram ABCD, AE =3x +7and CE=x+25.
What is AC?
O 68
O 34
O 18
09
D
C
B

Answers

In parallelogram ABCD, AE = CE since opposite sides of a parallelogram are congruent. Therefore, 3x + 7 = x + 25. Solving for x, we get x = 9. Substituting x = 9 into either AE or CE gives us the length of AC. So, AC = 3x + 7 = 3(9) + 7 = 34. Therefore, the answer is B. 34.

An insurance company offers an ordinary annuity that earns 6.5% interest compounded annually. A couple plans to make equal annual deposits into this account for 30 years and then make 20 equal annual withdrawals of €25,000, reducing the balance of the account to zero.

(i) Compute the value of the fund based on the withdrawals required. [5 marks]

(ii) Compute the amount of each deposit needed in order to maintain the fund. [5 marks]

(iii) Compute the total interest earned over the entire 50 years. [5 marks]​

Answers

Answer:

(i) To compute the value of the fund based on the withdrawals required, we can use the formula for the future value of an annuity due:

FV = P * ((1 + r)^n - 1) / r) * (1 + r)

where FV is the future value of the annuity, P is the annual payment, r is the interest rate per period, n is the total number of periods, and the extra (1 + r) factor is because the payments are made at the beginning of each period.

In this case, P = €25,000, r = 0.065, n = 20. We want to find the future value at the end of the 20-year period:

FV = 25000 * ((1 + 0.065)^20 - 1) / 0.065) * (1 + 0.065)

FV ≈ €743,704.96

Therefore, the value of the fund based on the withdrawals required is approximately €743,704.96.

(ii) To compute the amount of each deposit needed in order to maintain the fund, we can use the formula for the present value of an ordinary annuity:

PV = P * ((1 - (1 + r)^(-n)) / r)

where PV is the present value of the annuity, P is the annual payment, r is the interest rate per period, and n is the total number of periods.

In this case, PV = €743,704.96, r = 0.065, n = 20. We want to find the annual payment:

PV = P * ((1 - (1 + 0.065)^(-20)) / 0.065)

P ≈ €22,630.53

Therefore, the amount of each deposit needed in order to maintain the fund is approximately €22,630.53.

(iii) To compute the total interest earned over the entire 50 years, we can subtract the total deposits from the total withdrawals, and then subtract the initial balance. The total deposits are the annual deposit amount times the number of years (30), and the total withdrawals are the annual withdrawal amount times the number of years (20). The initial balance is the present value of the annuity that we calculated in part (ii).

Total deposits = €22,630.53 * 30 = €678,915.90

Total withdrawals = €25,000 * 20 = €500,000

Initial balance = €743,704.96

Total interest earned = Total withdrawals - Total deposits - Initial balance

Total interest earned = €500,000 - €678,915.90 - €743,704.96

Total interest earned ≈ -€922,620.86

Note that the negative sign indicates that the insurance company actually earned interest on this annuity, rather than the couple earning interest on their investment. This is because the withdrawals are greater than the deposits, and the interest rate earned by the insurance company is greater than the interest rate paid to the couple.

Step-by-step explanation:

how do you find the net of a rectangular prism

Answers

Answer:

The formula looks like this:

Surface Area = 2 l w + 2 l h + 2 h w ,

where SA = surface area, l = length, w = width, and h = height. In the rectangular prism net above, l = 8 inches, w = 5 inches, and h = 3 inches. Simply put these numbers into the formula and solve for surface area.

A belt runs a pulley at 80 revolutions per minute. Find the angular velocity of the pulley in radians per second.

Answers

A belt runs a pulley at 80 revolutions per minute.

(a) To find the angular speed of the pulley in radians per second, we can use the formula

ω = 2πf

Where

ω = angular speed in radians per second

f = frequency in Hertz

We know that the pulley rotates at a rate of 80 revolutions per minute. To convert this to a frequency in Hertz, we can divide by 60 seconds per minute

f = 80 rev/min ÷ 60 min/s = 4/3 Hz

Now we can plug in this value for f into the formula

ω = 2πf = 2π(4/3) = 8π/3 rad/sec

Therefore, the angular speed of the pulley is 8π/3 radians per second.

(b) To find the linear speed of the belt in centimeters per second, we can use the formula

v = rω

Where

v = linear speed in centimeters per second

r = radius of the pulley in centimeters

ω = angular speed in radians per second

We know the radius of the pulley is given in centimeters. Let's assume it is r cm. We just found the angular speed to be 8π/3 radians per second. Now we can plug in these values into the formula

v = rω = r(8π/3) = (8π/3)r cm/s

Therefore, the linear speed of the belt is (8π/3)r centimeters per second.

The given question is incomplete and the complete question is ''A belt runs a pulley at 80 revolutions per minute. Find the angular velocity of the pulley in radians per second. (b)Find the linear speed of the belt in centimeters per second''.

To know more about revolutions here

https://brainly.com/question/29707118

#SPJ1

Margarita fue a la tienda y compro una cartera y unos jeans por un
total de RD$3,250. Sabiendo que las cartera excede al jeans en
RD$970, ¿Cuántos pago margarita por cada artículo?
Cartera = RDS
Jeans = RD$
Escribir las respuestas numéricas y sin comas.
OK

Answers

The solution is , price of jeans = RD$ 1140 and, price of  handbag =

RD$ 2110.

Here, we have,

given that,

Margarita went to the store and bought a bag and some jeans for a total of RD$3,250.

Knowing that the handbag exceeds the jeans by RD$970,

now, we have to find that, how many do she pay for each article.

let, price of jeans = RD$ x

so, price of  handbag = RD$ (x +970)

ATQ, we get,

RD$ x + RD$ (x +970) = RD$3,250

or, RD$ 2x + 970 = RD$3,250

or, RD$ 2x = RD$ 2280

or, x = RD$ 1140

Hence, price of jeans = RD$ 1140 and, price of  handbag = RD$ 2110.

To earn more on addition click:

brainly.com/question/29560851

#SPJ1

A geography teacher assigns each student to write a report about one of the first 13 colonies. Students select the name of a colony by "blindly" drawing a colony's name from a bag. Once a colony has been drawn, it is not replaced.

What is the probability that the first student selects Pennsylvania and the second student selects Virginia? Round your answer to the nearest hundredth of a percent.

Answers

The probability that the first student selects Pennsylvania and the second student selects Virginia to the nearest hundredth of a percent is 0.64%.

Probability problem

The probability that the first student selects Pennsylvania is 1/13. Once Pennsylvania has been drawn, there are only 12 colonies left in the bag, so the probability that the second student selects Virginia is 1/12.

To find the probability that both events occur, we multiply the probabilities:

(1/13) x (1/12) = 1/156

To express this probability as a percentage, we multiply by 100:

(1/156) x 100 ≈ 0.64%

Therefore, the probability that the first student selects Pennsylvania and the second student selects Virginia is approximately 0.64%.

More on probability can be found here: https://brainly.com/question/30034780

#SPJ1

Need help. This please

Answers

The domain of the quadratic function in this problem is given as follows:

All real values.

How to obtain the domain of the function?

The domain of a function is the set of all the possible input values that can be assumed by the function.

On the graph, the domain of the function is given by the values of x of the function.

A quadratic function has no restrictions on the domain, hence it is defined by all the real values.

More can be learned about the domain of a function at https://brainly.com/question/10687170

#SPJ1

A curve, described by x2 + y2 + 6y = 0, has a point A at (−3, −3) on the curve.

Part A: What are the polar coordinates of A? Give an exact answer.

Part B: What is the polar form of the equation? What type of polar curve is this?

Part C: What is the directed distance when theta equals 4 pi over 3 question mark Give an exact answer.

Answers

a) The polar coordinates of point A are (√(18), π/4).

b) The curve is a circle centered at the origin with radius 6.

c) The directed distance is the value of r, which is 6 √(3).

To find the polar coordinates of point A on the curve, we need to convert the point from Cartesian to polar coordinates. The conversion formula is:

r = √(x² + y²)

θ = arctan(y/x)

Using the values of point A, we have:

r = √((-3)² + (-3)²) = √(18)

θ = arctan((-3)/(-3)) = arctan(1) = π/4

To find the polar form of the equation x² + y² + 6y = 0, we need to convert it from Cartesian to polar coordinates. The conversion formulae are:

x = r cos(θ)

y = r sin(θ)

Using these formulae, we can rewrite the equation as:

r² cos²(θ) + r² sin²(θ) + 6r sin(θ) = 0

Simplifying this equation, we get:

r = -6 sin(θ) / (1 - cos²(θ))

To find the directed distance when θ equals 4 π over 3, we need to substitute this value of θ into the polar equation we found in Part B. Doing so, we get:

r = -6 sin(4 π/3) / (1 - cos²(4 π/3))

r = -6(-√(3)/2) / (1 - (-1/2)²)

r = 6 √(3)

To know more about polar coordinates here

https://brainly.com/question/31422978

#SPJ1

Alice finds shirts on sale for $18.99.She buys twelve how much money does she spend?

Answers

Answer:

Well, if Alice buys 12 shirts which each cost $18.99 the equation would be 18.99 * 12 which = 227.88

Alice spent $227.88 on 12 shirts

Step-by-step explanation:

Answer:

Well, if Alice buys 12 shirts which each cost $18.99 the equation would be 18.99 * 12 which = 227.88

Step-by-step explanation:

What is the value of z?

Answers

Answer: z = 8

Step-by-step explanation:

The diagram shows us that 8z + 3z + 2 = 90

so we can say that: 11z = 88

therefore z = 8.

Note: diagrams can be misleading! this diagram technically shows us that 64 < 26!

You can use the notation P(A), read “the probability of event B, given event A” to write a

A. Probability distribution
B. Frequency table
C. Conditional probability
D. Cumulative probability

Answers

You can use the notation P(A), read “the probability of event B, given event A” to write a conditional probability. The correct answer is C.

Conditional probability refers to the probability of one event occurring given that another event has already occurred. In this case, we are interested in the probability of event B occurring given that event A has already occurred, and we can represent this using the notation P(B|A), where '|' means 'given'.

For example, let's say we are interested in the probability of getting a head on a coin toss (event B), given that the coin was flipped and landed on heads (event A). We could represent this using the notation P(B|A). The value of P(B|A) would be 1, because if the coin already landed on heads, then the probability of getting a head on the next flip is certain.

Conditional probability is an important concept in probability theory and is often used in real-world applications, such as predicting the likelihood of a disease given certain symptoms, or the probability of an event occurring given certain conditions.

The correct answer is C.

To learn more about probability click on,

https://brainly.com/question/29259732

#SPJ1

Forty people were asked their favorite kind of pizza. Thirty percent of the people surveyed chose sausage. How many people preferred sausage?

Answers

To find out how many people preferred sausage, you can start by multiplying the percentage who chose sausage, which is 30%, by the total number of people surveyed, which is 40.

So, 30% of 40 is:

0.30 x 40 = 12

Therefore, 12 people preferred sausage.

Answer: 12 people

Step-by-step explanation:

0.30 x 40 = 12

Helppp I need the ending numbers ?

Answers

The solution is, the simplification of the equation is :

4x^2 - 4 = (2x -2 ) (2x+2)

Here, we have,

given that,

the expression is:

4x^2 - 4

now, we have to simplify this.

we know that ,

The a^2 - b^2 formula is also known as "the difference of squares formula". The a square minus b square is used to find the difference between the two squares without actually calculating the squares.

It is one of the algebraic identities.

It is used to factorize the binomials of squares.

The a^2 - b^2 formula is given as:

a^2 - b^2 = (a - b) (a + b).

so, we have,

4x^2 - 4

= (2x)^2 - 2^2

= (2x -2 ) (2x+2)

Hence, The solution is, the simplification of the equation is :

4x^2 - 4 = (2x -2 ) (2x+2)

To learn more on Equation:

brainly.com/question/10413253

#SPJ1

Please help on this asap!! 65 Points

Answers

No, It is not possible for 75% of the people surveyed at the park to purchase any two combination of the treats unless 14 people from the 'No preference group" decide to pick a treat. The two best frozen treat to pick will be Ice-cream sandwich and frozen fruit bar. With these two, 174 people are guaranteed to purchase the product.

How do we find the 75% chance of people surveyed buying any two combination of frozen treats?

To find the 75% chance of people surveyed buying any two combination we say

58 + 95 + 79 + 17 = 250

75% of 250 = 187.5 which is 188

We need to see if 188 of people will pick up a treat based on the survey

58 + 95 = 153 < 188

95 + 79 = 174 < 188  however, if  14 people from no preference group purchase, then, this is the best combination.

58 + 79 =  137 < 188

The answer provided is based on the question below as seen in the picture;

Is it possible to select a combination of two frozen treats so that 75% of the people surveyed would be able to purchase their favorite? If so, which two types of frozen treats should you select? Use words and numbers to justify your answer

Find more exercises on survey population;

https://brainly.com/question/1398803

#SPJ1

The following blueprint of a kitchen has dimensions of 7 inches by 7 inches. The island has been highlighted in red.


The island's actual dimensions are 3 1/2 feet by 1 3/4 feet. If the scale of the blueprint is 1 inch = 2 feet, what are the dimensions of the island on the blueprint?

Answers

The dimensions of the island on the blueprint are 14 inches by 3.5 inches.

We have,

The actual dimensions of the island are 3 1/2 feet by 1 3/4 feet.

We need to find the dimensions of the island on the blueprint, given that the scale of the blueprint is 1 inch = 2 feet.

To convert the actual dimensions to the dimensions on the blueprint, we need to use the scale factor of 1 inch = 2 feet.

We can set up a proportion to relate the actual dimensions to the dimensions on the blueprint:

Actual dimension/blueprint dimension = scale factor

Let x be the length of the island on the blueprint.

Then we can set up the following proportion:

3.5 feet / (1.75 feet)

= x inches / 7 inches

Simplifying,

2 = x / 7

Multiplying both sides by 7, we get:

x = 14 inches

The length of the island on the blueprint is 14 inches.

Similarly, we can find the width of the island on the blueprint:

1.75 feet / 3.5 feet

= y inches / 7 inches

Simplifying, we have:

0.5 = y / 7

Multiplying both sides by 7, we get:

y = 3.5 inches

The width of the island on the blueprint is 3.5 inches.

Thus,

The dimensions of the island on the blueprint are 14 inches by 3.5 inches.

Learn more about expressions here:

https://brainly.com/question/3118662

#SPJ1

Need help pls and thank u! ​

Answers

Answer:

[tex]\sqrt[]{72}[/tex]

6.3141414141414...

[tex]\sqrt[4]{64}[/tex]

8.121 121 112 111...

Step-by-step explanation:

irrational numbers are real numbers that cannot be expressed as a ratio of integers.

example of irrational number [tex]\sqrt[]{2}[/tex]

example of rational number 2, 3, -4, etc.

Brooklyn and Matthew both recently got a job and want to start a savings account to earn interest on money they save. Brooklyn's paycheck for was for $625 net pay-after taxes. the bank her parents use offer savings account compounded monthly with an interest rate of 2.5%. Matthews paycheck was also for $446 net pay. his parents know of a savings account that earns 4% compounded quarterly.
a. use the compounding interest rate formula to identify who will have more money after two years (assuming no additional deposits or withdrawals are made)
b. use the Desmos graphing calculator to graph Brooklyn's equation and Matthew’s equation on the same graph. identify the points of intersection and interpret the results.

WHAT MISTAKES DID THE STUDENTS MAKE??

Answers

After two years, Brooklyn will have more money in her savings account than Matthew.

Brooklyn's savings account grows at a slower rate than Matthew's initially but eventually catches up and surpasses Matthew's account due to the higher interest rate and monthly compounding.

a.

[tex]A = P(1 + r/n)^{nt}[/tex]

where A is the final amount, P is the initial principal (or net pay), r is the interest rate (as a decimal), n is the number of times the interest is compounded per year, and t is the number of years.

For Brooklyn saving account,

P = $625, r = 0.025, n = 12 (monthly compounding), and t = 2.

A = $625(1 + 0.025/12)^(12*2) = $686.71

For Matthew saving account,

P = $446, r = 0.04, n = 4 (quarterly compounding), and t = 2.

A = $446(1 + 0.04/4)^(4*2) = $506.84

b.

To graph Brooklyn's equation and Matthew's equation on the same graph, we can use the following equations:

Brooklyn's equation: A = 625(1 + 0.025/12)^(12t)

Matthew's equation: A = 446(1 + 0.04/4)^(4t)

Now,

The resulting graph will show the growth of each saving account over time.

The points of intersection on the graph represent the times when the two savings accounts have the same amount of money.

From the graph, we can see that the two accounts intersect at around 11 months and 23 months.

We can say that Brooklyn's savings account grows at a slower rate than Matthew's initially, but eventually catches up and surpasses Matthew's account due to the higher interest rate and monthly compounding.

Thus,

After two years, Brooklyn will have more money in her savings account than Matthew.

Brooklyn's savings account grows at a slower rate than Matthew's initially but eventually catches up and surpasses Matthew's account due to the higher interest rate and monthly compounding.

Learn more about saving accounts here:

https://brainly.com/question/13155407

#SPJ1

math hw for tonight
help solve this problem! Thank you!
ap cal bc

Answers

Answer:

first option

Step-by-step explanation:

differentiate using the power rule

[tex]\frac{d}{dx}[/tex] (a[tex]x^{n}[/tex] ) = na[tex]x^{n-1}[/tex]

then

[tex]\frac{dy}{dx}[/tex] = [tex]\frac{dy}{dt}[/tex] × [tex]\frac{dt}{dx}[/tex] = [tex]\frac{\frac{dy}{dt} }{\frac{dx}{dt} }[/tex]

y = t² + 4t

[tex]\frac{dy}{dt}[/tex] = 2t + 4

x = t - 3

[tex]\frac{dx}{dt}[/tex] = 1

then

[tex]\frac{dy}{dx}[/tex] = [tex]\frac{2t+4}{1}[/tex] = 2t + 4

Answer:

2t + 4

Step-by-step explanation:

A parametric equation is one where x and y are defined separately in terms of a third variable (often the parameter t).

To find dy/dx from parametric equations, differentiate each equation with respect to the parameter t, then use the chain rule:

[tex]\boxed{\dfrac{\text{d}y}{\text{d}x}=\dfrac{\text{d}y}{\text{d}t} \times \dfrac{\text{d}t}{\text{d}x}}[/tex]

Differentiate the two parametric equations with respect to t:

[tex]x=t-3 \implies \dfrac{\text{d}x}{\text{d}t}=1[/tex]

[tex]y=t^2+4t \implies \dfrac{\text{d}y}{\text{d}t}=2t+4[/tex]

Use the chain rule to combine them:

[tex]\begin{aligned}\implies \dfrac{\text{d}y}{\text{d}x}&=\dfrac{\text{d}y}{\text{d}t} \times \dfrac{\text{d}t}{\text{d}x}\\\\&=(2t+4) \times \dfrac{1}{1}\\\\&=2t+4\end{aligned}[/tex]

Therefore:

[tex]\boxed{\dfrac{\text{d}y}{\text{d}x}=2t+4}[/tex]

Which equation represents a direct variation?
Oy - 4x = 8
Oy + 2 = 7x
Oy - 3x = 0
O y = 5x - 2

Answers

The only equation that represents a direct variation is: y - 3x = 0

How to identify direct variation?

Direct Variation is defined as the relationship that exists between two variables in which one is a constant multiple of the other. For example, when one variable changes the other, then they are said to be in proportion. If b is directly proportional to a the equation is of the form b = ka (where k is a constant).

Making y the subject in each of the options gives:

A) y = 4x + 8

B) y = 7x - 2

C) y = 3x

D) y = 5x - 2

Looking at them, the only one where there is a relationship that exists between two variables in which one is a constant multiple of the other is option C

Read more about Direct variation at: https://brainly.com/question/6499629

#SPJ1

construct a binomial whose gcf is 7a^3

Answers

The binomial whose greatest common factor is 7a³ is 14a⁴ + 35a³.

Given, the a³ is not common in A and C.

Now, in 14a⁴ + 35a³

14a⁴ = 2 x 7 x a³ x a

35a³ = 7 x 5 x a³

So, the common factors of 14 and 35 is 7.

Then, 14a⁴ + 35a³

= ( 2 x 7 x a³ x a) + (  5 x 7 x a³)

= 7a³ (2a + 5)

Learn more about greatest common factor here:

brainly.com/question/11221202

#SPJ1

Can I get some help in this question?

What value of x satisfies the equation (7q^3x)^3=343q^36

Answers

4 is the value of the variable x.

What is algebraic Expression?

Any mathematical statement that includes numbers, variables, and an arithmetic operation between them is known as an expression or algebraic expression. In the phrase 4m + 5, for instance, the terms 4m and 5 are separated from the variable m by the arithmetic sign +.

We can simplify the left side of the equation by using the properties of exponents:

[tex](7q^{3x})^3 = 7^3 * (q^{3x})^3 = 343q^{9x}\\\\343q^{9x} = 343q^{36}\\\\q^{9x} = q^{36[/tex]

Now we can use the property that if [tex]a^b = a^c[/tex], then b = c. Therefore:

9x = 36

Dividing both sides by 9, we get:

x = 4

Therefore, the value of x that satisfies the equation is 4.

Learn more about algebraic Expression here:

https://brainly.com/question/953809

#SPJ1

The distance between cities A and B on a map is 12.5 in. The distance from city B to city C, is 8.5 in, and the distance from C to A is 16.25 in. If the bearing
from A to B is N75°E, find the bearing from C to 4. Round to the nearest tenth of a degree.
16.25 in
12.5 in
8.5 in
The bearing from city C to city 4 is approximately (Choose one) (Choose one)

Answers

The bearing of C to A is 239⁰.

What is the bearing of C to A?

The bearing of C to A is calculated by finding the value of angle C using cosine rule since we know the value of all the sides of the triangle.

AB² = AC² + CB² - 2(AC)(CB) cosC

12.5² = 16.25² + 8.5² - 2(16.25 x 8.5) x cos C

156.25 = 336.3125 - 276.25cosC

276.25cosC = 180.06

cosC = 180.06/276.25

cos (C) = 0.6518

C = cos⁻(0.6518)

C = 49.3⁰

The value of angle A is calculated as follows;

Sin A/CB = Sin C/AB

sin A/8.5 = sin 49.3/12.5

sin A = 8.5 [sin 49.3/12.5]

sin A = 0.5157

A = sin⁻¹ (0.5157)

A = 31⁰

The bearing of C to A is calculated as;

= 270⁰ - 31⁰

= 239⁰

Learn more about bearing here: https://brainly.com/question/28782815

#SPJ1

Answer all questions please

Answers

The value of f(1) is 3.

An estimate of the value of f(-1) is -0.2.

The values of x for which f(x) = 1 are: x = (0, 3).

The value of x such that f(x) = 0 is x = -0.5.

The domain of f is {-2, 4}.

The range of f is {-1, 3}.

The interval over which f is increasing is {-2, 1}.

What is a domain?

In Mathematics and Geometry, a domain is the set of all real numbers for which a particular function is defined.

Furthermore, the vertical extent of any graph of a function represents all range values and they are always read and written from smaller to larger numerical values, and from the bottom of the graph to the top.

By critically observing the graph of the function shown in the image attached above, we can reasonably and logically deduce the following domain and range:

Domain = {-2, 4}.

Range = {-1, 3}.

In conclusion, we can logically deduce that this function is increasing over the [-2, 1] and decreasing over the interval [1, 4].

Read more on domain here: brainly.com/question/17440903

#SPJ1

geometry pls help fast 13 and 14

Answers

Answer:

13: (2,3) 14: A. Yes, 90, 180, 270 B. No C. No

Step-by-step explanation:

13. Right: -2+4, Down: 5-2

14. If it can be rotated and be similar, it has rotational.

Question 1 (1 point)
Write an inequality for the sentence.
The stadium held less than 25,000.
B
O a
O b
9ba580e611107c96c9efb866417dc160.webm 64 KB
s> 25,000
$≤25,000
Oc $<25,000

Answers

The inequality that represents the sentence "The stadium held less than 25,000 people" is given as follows:

c < 25,000.

What are the inequality symbols?

The four inequality symbols, along with their meaning on the number line and the coordinate plane, are presented as follows:

> x: the amount is greater than x -> the number is to the right of x with an open dot at the number line. -> points above the dashed horizontal line y = x on the coordinate plane.< x: the amount is less than x. -> the number is to the left of x with an open dot at the number line. -> points below the dashed horizontal line y = x on the coordinate plane.≥ x: the amount is at least x. -> the number is to the right of x with a closed dot at the number line. -> points above the solid vertical line y = x on the coordinate plane.≤ the amount is at most x. -> the number is to the left of x with a closed dot at the number line. -> points above the dashed vertical line y = x on the coordinate plane.

The amount is less than in this problem, hence the symbol is given as follows:

<.

As the amount is less than 25000, the inequality is given as follows:

c < 25,000.

More can be learned about inequalities at brainly.com/question/25275758

#SPJ1

Other Questions
What is the generic name for Colcrys? Allopurinol Colchicine Pegioticase Probenecid Look at picture! It is all written there A growth medium that distinguishes among different groups of bacteria on the basis of their biological characteristics is called a __________ medium. A. selective B. differential C. enrichment D. transport PLEASE HELP ME Using 2-3 sentences describe Natural Selection. Use and underline/highlight the terms adaptations, traits, diversity, mutations,environment, population, and natural selection. One monoclonal antibody drug can be used to attack a variety of different types of cancer.TrueFalse If a solid has 6 faces and 12 edges, how many vertices does it have? A. 10 vertices B. 9 vertices C. 7 vertices D. 8 vertices tertiary syphillis can lead to damage to internal organs such as in what 7 organs? (BNEHBLBJ) details about the composition of intangible assets are located in the summary of significant account policies 2 What idea does the speaker convey in line 16 when he says the sea is alsoearth?A) Symbolic language is often too vague.B) The sea and the earth are both symbols.C) Seawater is often mixed with earth.D) Rising seas are diminishing the earth. A 35-ft long solid steel rod is subjected to a load of 8,000 lb. This load causes the rod to stretch 0.266 in. The modulus of elasticity of the steel is 30,000,000 psi. Determine the diameter of the rod (precision of 0.00) 17. the millers incurred childcare expenses for their daughter, izzy, of $9,000 in 2022 in order for them to work. these expenses were paid to sherry ton (ssn: 482-93-0923), who lives at 4532, main street, long beach, ca 90223. Alicia bitman,age 30,plans to purchase a $50,000 whole-life policy what is the annual premium How did The Gates by Christo and Jeanne-Claude affect the site? The Gates, site-specific work of art with a panel of deep saffron-colored nylon fabric hung from each of the numerous vinyl gates along the pathways in Central Park in New York City, by Christo and Jeanne-Claude Question 1 options:The experience of the park with The Gates was so different that some people saw the park in a new way.The permanent installation at the site remains a source of controversy among New Yorkers.The traffic along nearby roads was blocked for days to protect the artwork from people and cars.The landscape was permanently altered from the graffiti left in the wake of the project. 5) For plant expansion, a communication company wishes to accumulate P50 M by investing equal amount each semi-annual for 50 years. If money is worth 20% nominal, how much must be invested semi-annually? a) P 3, 205, 616.80 b) P 2, 750, 605.80 c) P 3, 416, 205.90 d) P 2, 509, 815.80 in (x-2)+in(x+1)=2x = -2.6047x = 4.2312x = 3.652x = 3.6047 Find the value of sin N rounded to the nearest hundredth, if necessary Layer 5 of the OSI model is called:1) Session layer2) Application layer3) Transport layer4) Presentation layer Question 5 (1 point)According to McEwan (2015), _________ suggests the closer people are, the morechannels they use to communicate with each other.Weak tiesIntersticesMultiplexityAugmented flesh meet On January 1, 2017, Blade Inc. purchased a machine for $104,000. Blade depreciated the machine with the straight-line depreciation method over a useful life of 10 years, using a residual value of $4,000. At the beginning of 2019, a major overhaul, costing $30,000, was made. After the overhaul, the machine's residual value is estimated to be $6,000, and the machine is expected to have a remaining useful life of 12 years. Required: Determine the depreciation expense for 2019. Who did you nominate to receive the humanitarian award? Why did you nominate this person? Discuss your peers' nominations. Cast your vote.